Difference between revisions of "2016 AMC 10B Problems/Problem 16"

(Solution)
(Solution)
Line 14: Line 14:
 
<math>S=\frac{a_1}{1-r}</math>
 
<math>S=\frac{a_1}{1-r}</math>
 
where <math>a_1</math> is the first term and <math>r</math> is the ratio whose absolute value is less than 1.
 
where <math>a_1</math> is the first term and <math>r</math> is the ratio whose absolute value is less than 1.
We know that the second term is the first term multiplied by the ratio.  
+
We know that the second term(<math>1</math>) is the first term multiplied by the ratio.  
 
In other words:
 
In other words:
 
<math>a_1 \cdot r= a_2</math>,
 
<math>a_1 \cdot r= a_2</math>,
Line 21: Line 21:
 
<math>a_1=\frac{1}{r}</math>.
 
<math>a_1=\frac{1}{r}</math>.
 
Thus the sum is the following:
 
Thus the sum is the following:
<math>S=\frac{\frac{1}{r}}{1-r}</math>
+
<math>S=\frac{\frac{1}{r}}{1-r}</math>.
 
We can multiply <math>r</math> to both sides of the numerator and denominator.
 
We can multiply <math>r</math> to both sides of the numerator and denominator.
<math>S=\frac{1}{r-r^2}</math>
+
<math>S=\frac{1}{r-r^2}</math>.
 
Since we want the minimum value of this expression, we want the maximum value for the denominator which is a quadratic of the form
 
Since we want the minimum value of this expression, we want the maximum value for the denominator which is a quadratic of the form
<math>-r^2+r</math>
+
<math>-r^2+r</math>.
 
The maximum value of a quadratic with negative <math>a</math> is <math>\frac{-b}{2a}</math>.
 
The maximum value of a quadratic with negative <math>a</math> is <math>\frac{-b}{2a}</math>.
<math>S=\frac{-(1)}{2(-1)}=\frac{1}{2}</math>
+
<math>S=\frac{-(1)}{2(-1)}=\frac{1}{2}</math>.
 
Plugging 1/2 in, we get:
 
Plugging 1/2 in, we get:
 
<math>S=\frac{1}{\frac{1}{2}}=2</math>, <math>B</math>.
 
<math>S=\frac{1}{\frac{1}{2}}=2</math>, <math>B</math>.

Revision as of 14:12, 21 February 2016

Problem

The sum of an infinite geometric series is a positive number $S$, and the second term in the series is $1$. What is the smallest possible value of $S?$

$\textbf{(A)}\ \frac{1+\sqrt{5}}{2} \qquad \textbf{(B)}\ 2 \qquad \textbf{(C)}\ \sqrt{5} \qquad \textbf{(D)}\ 3 \qquad \textbf{(E)}\ 4$


Solution

The sum of an infinite geometric series is of the form: $S=\frac{a_1}{1-r}$ where $a_1$ is the first term and $r$ is the ratio whose absolute value is less than 1. We know that the second term($1$) is the first term multiplied by the ratio. In other words: $a_1 \cdot r= a_2$, $a_2=1$ (given), $a_1 \cdot r=1$, and $a_1=\frac{1}{r}$. Thus the sum is the following: $S=\frac{\frac{1}{r}}{1-r}$. We can multiply $r$ to both sides of the numerator and denominator. $S=\frac{1}{r-r^2}$. Since we want the minimum value of this expression, we want the maximum value for the denominator which is a quadratic of the form $-r^2+r$. The maximum value of a quadratic with negative $a$ is $\frac{-b}{2a}$. $S=\frac{-(1)}{2(-1)}=\frac{1}{2}$. Plugging 1/2 in, we get: $S=\frac{1}{\frac{1}{2}}=2$, $B$.

See Also

2016 AMC 10B (ProblemsAnswer KeyResources)
Preceded by
Problem 15
Followed by
Problem 17
1 2 3 4 5 6 7 8 9 10 11 12 13 14 15 16 17 18 19 20 21 22 23 24 25
All AMC 10 Problems and Solutions

The problems on this page are copyrighted by the Mathematical Association of America's American Mathematics Competitions. AMC logo.png